Difference between revisions of "2020 AMC 10A Problems/Problem 16"

(Video Solution)
(5 intermediate revisions by 4 users not shown)
Line 12: Line 12:
 
size(10cm);
 
size(10cm);
 
draw((0,0)--(1,0)--(1,1)--(0,1)--cycle);
 
draw((0,0)--(1,0)--(1,1)--(0,1)--cycle);
filldraw((arc((0,0), 0.4, 0, 90))--(0,0)--cycle, gray);
+
filldraw((arc((0,0), 0.3989, 0, 90))--(0,0)--cycle, gray);
draw(arc((1,0), 0.4, 90, 180));
+
draw(arc((1,0), 0.3989, 90, 180));
filldraw((arc((1,0), 0.4, 90, 180))--(1,0)--cycle, gray);
+
filldraw((arc((1,0), 0.3989, 90, 180))--(1,0)--cycle, gray);
draw(arc((1,1), 0.4, 180, 270));
+
draw(arc((1,1), 0.3989, 180, 270));
filldraw((arc((1,1), 0.4, 180, 270))--(1,1)--cycle, gray);
+
filldraw((arc((1,1), 0.3989, 180, 270))--(1,1)--cycle, gray);
draw(arc((0,1), 0.4, 270, 360));
+
draw(arc((0,1), 0.3989, 270, 360));
filldraw(arc((0,1), 0.4, 270, 360)--(0,1)--cycle, gray);
+
filldraw(arc((0,1), 0.3989, 270, 360)--(0,1)--cycle, gray);
 
</asy>
 
</asy>
  
Line 45: Line 45:
 
As above, we find that we need to estimate <math>d = \frac{1}{\sqrt{2\pi}}</math>.  
 
As above, we find that we need to estimate <math>d = \frac{1}{\sqrt{2\pi}}</math>.  
  
Note that we can approximate <math>2\pi \approx 6.28 \approx 6.25</math> and so <math>\frac{1}{\sqrt{2\pi}}</math> <math>\approx \frac{1}{\sqrt{6.25}}=\frac{1}{2.5}=0.4</math>.
+
Note that we can approximate <math>2\pi \approx 6.28318 \approx 6.25</math> and so <math>\frac{1}{\sqrt{2\pi}}</math> <math>\approx \frac{1}{\sqrt{6.25}}=\frac{1}{2.5}=0.4</math>.
  
 
And so our answer is <math>\boxed{\textbf{(B) } 0.4}</math>.
 
And so our answer is <math>\boxed{\textbf{(B) } 0.4}</math>.
  
 
~Silverdragon
 
~Silverdragon
 +
 
==Video Solution==
 
==Video Solution==
 +
 +
Education, The Study of Everything
 +
 +
https://youtu.be/napCkujyrac
 +
 
https://youtu.be/RKlG6oZq9so
 
https://youtu.be/RKlG6oZq9so
  
Line 59: Line 65:
 
{{AMC10 box|year=2020|ab=A|num-b=15|num-a=17}}
 
{{AMC10 box|year=2020|ab=A|num-b=15|num-a=17}}
 
{{AMC12 box|year=2020|ab=A|num-b=15|num-a=17}}
 
{{AMC12 box|year=2020|ab=A|num-b=15|num-a=17}}
 +
 +
[[Category: Introductory Algebra Problems]]
 
{{MAA Notice}}
 
{{MAA Notice}}

Revision as of 11:52, 7 November 2020

The following problem is from both the 2020 AMC 12A #16 and 2020 AMC 10A #16, so both problems redirect to this page.

Problem

A point is chosen at random within the square in the coordinate plane whose vertices are $(0, 0), (2020, 0), (2020, 2020),$ and $(0, 2020)$. The probability that the point is within $d$ units of a lattice point is $\tfrac{1}{2}$. (A point $(x, y)$ is a lattice point if $x$ and $y$ are both integers.) What is $d$ to the nearest tenth$?$

$\textbf{(A) } 0.3 \qquad \textbf{(B) } 0.4 \qquad \textbf{(C) } 0.5 \qquad \textbf{(D) } 0.6 \qquad \textbf{(E) } 0.7$

Solution 1

Diagram

[asy] size(10cm); draw((0,0)--(1,0)--(1,1)--(0,1)--cycle); filldraw((arc((0,0), 0.3989, 0, 90))--(0,0)--cycle, gray); draw(arc((1,0), 0.3989, 90, 180)); filldraw((arc((1,0), 0.3989, 90, 180))--(1,0)--cycle, gray); draw(arc((1,1), 0.3989, 180, 270)); filldraw((arc((1,1), 0.3989, 180, 270))--(1,1)--cycle, gray); draw(arc((0,1), 0.3989, 270, 360)); filldraw(arc((0,1), 0.3989, 270, 360)--(0,1)--cycle, gray); [/asy]

Diagram by MathandSki Using Asymptote

Note: The diagram represents each unit square of the given $2020 * 2020$ square.

Solution

We consider an individual one-by-one block.

If we draw a quarter of a circle from each corner (where the lattice points are located), each with radius $d$, the area covered by the circles should be $0.5$. Because of this, and the fact that there are four circles, we write

\[4 * \frac{1}{4} * \pi d^2 = \frac{1}{2}\]

Solving for $d$, we obtain $d = \frac{1}{\sqrt{2\pi}}$, where with $\pi \approx 3$, we get $d = \frac{1}{\sqrt{6}}$, and from here, we simplify and see that $d \approx 0.4 \implies \boxed{\textbf{(B) } 0.4.}$ ~Crypthes

$\textbf{Note:}$ To be more rigorous, note that $d<0.5$ since if $d\geq0.5$ then clearly the probability is greater than $\frac{1}{2}$. This would make sure the above solution works, as if $d\geq0.5$ there is overlap with the quartercircles. $\textbf{- Emathmaster}$


Solution 2

As in the previous solution, we obtain the equation $4 * \frac{1}{4} * \pi d^2 = \frac{1}{2}$, which simplifies to $\pi d^2 = \frac{1}{2} = 0.5$. Since $\pi$ is slightly more than $3$, $d^2$ is slightly less than $\frac{0.5}{3} = 0.1\bar{6}$. We notice that $0.1\bar{6}$ is slightly more than $0.4^2 = 0.16$, so $d$ is roughly $\boxed{\textbf{(B) } 0.4}.$ ~emerald_block

Solution 3 (Estimating)

As above, we find that we need to estimate $d = \frac{1}{\sqrt{2\pi}}$.

Note that we can approximate $2\pi \approx 6.28318 \approx 6.25$ and so $\frac{1}{\sqrt{2\pi}}$ $\approx \frac{1}{\sqrt{6.25}}=\frac{1}{2.5}=0.4$.

And so our answer is $\boxed{\textbf{(B) } 0.4}$.

~Silverdragon

Video Solution

Education, The Study of Everything

https://youtu.be/napCkujyrac

https://youtu.be/RKlG6oZq9so

~IceMatrix

See Also

2020 AMC 10A (ProblemsAnswer KeyResources)
Preceded by
Problem 15
Followed by
Problem 17
1 2 3 4 5 6 7 8 9 10 11 12 13 14 15 16 17 18 19 20 21 22 23 24 25
All AMC 10 Problems and Solutions
2020 AMC 12A (ProblemsAnswer KeyResources)
Preceded by
Problem 15
Followed by
Problem 17
1 2 3 4 5 6 7 8 9 10 11 12 13 14 15 16 17 18 19 20 21 22 23 24 25
All AMC 12 Problems and Solutions

The problems on this page are copyrighted by the Mathematical Association of America's American Mathematics Competitions. AMC logo.png